Skip to main content
Mathematics LibreTexts

1.2: Ordinary Differential Equations

  • Page ID
    2130
  • \( \newcommand{\vecs}[1]{\overset { \scriptstyle \rightharpoonup} {\mathbf{#1}} } \) \( \newcommand{\vecd}[1]{\overset{-\!-\!\rightharpoonup}{\vphantom{a}\smash {#1}}} \)\(\newcommand{\id}{\mathrm{id}}\) \( \newcommand{\Span}{\mathrm{span}}\) \( \newcommand{\kernel}{\mathrm{null}\,}\) \( \newcommand{\range}{\mathrm{range}\,}\) \( \newcommand{\RealPart}{\mathrm{Re}}\) \( \newcommand{\ImaginaryPart}{\mathrm{Im}}\) \( \newcommand{\Argument}{\mathrm{Arg}}\) \( \newcommand{\norm}[1]{\| #1 \|}\) \( \newcommand{\inner}[2]{\langle #1, #2 \rangle}\) \( \newcommand{\Span}{\mathrm{span}}\) \(\newcommand{\id}{\mathrm{id}}\) \( \newcommand{\Span}{\mathrm{span}}\) \( \newcommand{\kernel}{\mathrm{null}\,}\) \( \newcommand{\range}{\mathrm{range}\,}\) \( \newcommand{\RealPart}{\mathrm{Re}}\) \( \newcommand{\ImaginaryPart}{\mathrm{Im}}\) \( \newcommand{\Argument}{\mathrm{Arg}}\) \( \newcommand{\norm}[1]{\| #1 \|}\) \( \newcommand{\inner}[2]{\langle #1, #2 \rangle}\) \( \newcommand{\Span}{\mathrm{span}}\)\(\newcommand{\AA}{\unicode[.8,0]{x212B}}\)

    Set

    $$E(v)=\int_a^bf(x,v(x),v'(x))\ dx\]

    and for given \(u_a,\ u_b\in{\mathbb R}\)

    $$V=\{v\in C^2[a,b]:\ v(a)=u_a,\ v(b)=u_b\},\]

    where \(y\) and \(f\) is sufficiently regular. One of the basic problems in the calculus of variation is

    (P) \(\min_{v\in V}E(v)\).

    Euler equation

    Let \(u\in V\) be a solution of (P), then

    $$\frac{d}{dx}f_{u'}(x,u(x),u'(x))=f_u(x,u(x),u'(x))\]

    in \((a,b)\).

    Exercise \(\PageIndex{1}\): Proof

    For fixed \(\phi\in C^2[a,b]\) with \(\phi(a)=\phi(b)=0\) and real \(\epsilon\), \(|\epsilon|<\epsilon_0\), set \(g(\epsilon)=E(u+\epsilon \phi)\). Since \(g(0)\le g(\epsilon)\) it follows \(g'(0)=0\). Integration by parts in the formula for \(g'(0)\) and the following basic lemma in the calculus of variations imply Euler's equation.

    Admissible Variations

    Figure 1.2.1.1: Admissible Variations

    Basic lemma in the calculus of variations. Let \(h\in C(a,b)\) and

    $$\int_a^bh(x)\phi(x)\ dx=0$$

    for all \(\phi\in C_0^1(a,b)\). Then \(h(x)\equiv0\) on \((a,b)\).

    Proof. Assume \(h(x_0)>0\) for an \(x_0\in (a,b)\), then there is a \(\delta>0\) such that \((x_0-\delta,x_0+\delta)\subset(a,b)\) and \(h(x)\ge h(x_0)/2\) on \((x_0-\delta,x_0+\delta)\).
    Set

    $$
    \phi(x)
    =\left\{\begin{array}{r@{\quad\mbox{if}\quad}l}
    \left(\delta^2-|x-x_0|^2\right)^2 & x\in(x_0-\delta,x_0+\delta)\\
    0 & x\in (a,b)\setminus[x_0-\delta,x_0+\delta]
    \end{array} \right. .
    \]

    Thus \(\phi\in C_0^1(a,b)\) and

    $$\int_a^b h(x)\phi(x)\ dx\ge \frac{h(x_0)}{2}\int_{x_0-\delta}^{x_0+\delta}\phi(x)\ dx>0,\]

    which is a contradiction to the assumption of the lemma.

    \(\Box\)

    Contributors and Attributions


    This page titled 1.2: Ordinary Differential Equations is shared under a not declared license and was authored, remixed, and/or curated by Erich Miersemann.

    • Was this article helpful?